LSAT and Law School Admissions Forum

Get expert LSAT preparation and law school admissions advice from PowerScore Test Preparation.

 Administrator
PowerScore Staff
  • PowerScore Staff
  • Posts: 8917
  • Joined: Feb 02, 2011
|
#23352
Complete Question Explanation

Parallel Flaw-CE. The correct answer choice is (C)

This Parallel question uses causal reasoning, but concludes that the cause does not work. Antitheft devices do not have their intended cause of deterring theft. We should first look for an answer that involves causal reasoning and concludes that the cause does not produce the intended effect.

Answer choice (A): Although this answer choice seems to involve causal reasoning, it does not conclude that a cause does not produce an intended effect based on these surveys. Its conclusion does not match the stimulus's conclusion at all.

Answer choice (B): There does not appear to be any cause and effect reasoning in this answer choice. The conclusion is about who the library is intended to serve based on its collection. We do not know if this intention is successful or not (which is what the conclusion of the stimulus concerns). Again, if you Double the Conclusion, you will see that this conclusion does not match.

Answer choice (C): This is the correct answer choice, as it follows the pattern of reasoning present in the stimulus. It presents a study (notice that the parts of the argument need to match — the stimulus presents statistics) that seems to contradict common sense. Then, it concludes that one effect libraries may be intended or thought to produce does not occur.

Answer choice (D): The conclusion here: ("children who do not like to read usually have perfect vision") besides being ridiculous, does not match the "do not protect" of the stimulus at all. Although this is extremely flawed reasoning, it does not match the reasoning of the stimulus.

Answer choice (E): This answer choice postulates a cause and effect relationship, but it never claims that an intended or commonsense effect does not occur like the conclusion of the stimulus does.
 halincandenza
  • Posts: 1
  • Joined: Jul 30, 2017
|
#37804
I'm still a bit confused on this question; I interpreted the flaw as being a confusion of causation and correlation; maybe cars with auto-theft devices are stolen more, which seems more in line with D or E.
 AthenaDalton
PowerScore Staff
  • PowerScore Staff
  • Posts: 296
  • Joined: May 02, 2017
|
#37838
Hi halincadenza,

You're right that the author is confusing correlation and causation. In the example in the prompt, he reasons that since cars with antitheft devices are more likely to be stolen, the antitheft devices don't work. He's failing to take into account the fact that these two factors are actually both effects of a single cause: expensive cars are both more likely to have antitheft devices and more likely to be targeted by car thieves. The cause (owning an expensive car) produces both effects (installing an anti-theft device in the car and having the car stolen).

Answer choice (C) mirrors this reasoning. The author says that people who use libraries frequently are also more likely to buy a lot of books. He then concludes on this basis that using libraries doesn't reduce the number of books an individual buys. The flaw is that he's missing the independent cause behind both factors: individuals who both use libraries frequently and purchase books are probably bookworms who just read a lot more than everyone else. An independent cause (being a bookworm) is producing both effects (using the library frequently and buying lots of books).

Answer choice (D) doesn't parallel the flaw in the stimulus because answer choice (D) is actually an example of accurate reasoning.

Answer choice (E) also depends on causal reasoning, but unlike in the stimulus it doesn't fail to account for an obvious alternate cause that produce the effect of both supporting free libraries and free universities.
 Tajadas
  • Posts: 63
  • Joined: Apr 11, 2020
|
#83083
Hello,

I got this question right, but I had a lot of trouble prephrasing the answer. I saw many problems with the stim-- maybe the cars with the antitheft devices are left in more dangerous areas. Maybe there are just more cars with antitheft devices, so more get stolen. Maybe more cars with antitheft are stolen, but even more would be if they didn't have antitheft. Because there were so many issues with the stim, I had a lot of trouble distilling the flaw and prephrasing it. I eventually awkwardly took the "maybe even more cars would be stolen without antitheft" error and translated it to "maybe regular library users would purchase even more books without libraries". I got the question right, but compared with the above prephrase, it was a lot harder than it needed to be. How did you make such a simple but effective prephrase?
 Adam Tyson
PowerScore Staff
  • PowerScore Staff
  • Posts: 5153
  • Joined: Apr 14, 2011
|
#83414
It can be helpful sometimes to approach Flaw questions like Weaken questions, Tajadas. If you go on the attack, you will be identifying and taking advantage of the Flaw. Here, your last prephrase turns out to be a great Weaken answer - what if those cars with antitheft devices would have been stolen even more without those devices? What this prephrase does is contradict the conclusion: they say the devices don't help, and you respond with "well what if they DO help?"

From there you are just a short step away from identify the flaw, which is that the argument fails to consider that maybe the devices are helping, that maybe things would have been even worse without them. Failing to acknowledge the weakness in the argument is a good way to describe just about any flaw!

Think about simply contradicting the conclusion in the stimulus, and you are going to find yourself very close to having a great prephrase for a Weaken or Flaw question. Don't overthink it - just think "nope, you're wrong" and go from there!
User avatar
 catherineshi99
  • Posts: 9
  • Joined: May 23, 2021
|
#88671
Hi there,

I don't understand how this argument is flawed. The author is not saying that anti-theft devices increase the likelihood that a car will be broken into, they are just saying that anti-theft devices do not protect against break-ins, which is true as stated in the previous statement.

If the anti-theft devices did successfully protect against theft, then no car with an anti-theft device would ever be broken into, but because they are broken into as stated in the first sentence, clearly anti-theft devices do not successfully protect against theft, which is the author's conclusion.

Thanks!
User avatar
 atierney
PowerScore Staff
  • PowerScore Staff
  • Posts: 215
  • Joined: Jul 06, 2021
|
#88704
Hi Catherine,

You seem like you have very high standards for both success rates of commercial products and the utility of such products! I understand where you're coming from, the idea is that well, how can you say something works (the anti-theft device protecting against thieves), if it has a higher theft rate!

I think one way perhaps, of looking at this would be by analogy. What comes to mind for me is another instance (also tested somewhat regularly on the LSAT), where the "rate of success" doesn't really tell the whole story. Consider the brain surgeon. Oh what a lovely, lovely job, only operating in the most dire times, with patients who are either in a comatose or are clinging to life... you could imagine the success rate (judged literally here by life and death) to be well, not great. And this is even in spite of the fact that our imaginary brain surgeon was Harvard educated and generally considered to be the best brain surgeon in the world! Still, the common dental surgeon's "success rate" (if say, measured by life and death) would dwarf our poor hero's.

Hopefully, the takeaway is clear. Stats, they don't tell the whole story. Often times, they're a sword that can wielded by whomever to make whatever cuts they like. (other times, they're just damn lies!). And here, the sole "premise" or evidence used to indicate a rather strong claim, that anti-theft devices simply don't work (at all) is the relative rate of cars broken into using them as compared to the similar rate in cars that have not. Remember, our brain surgeon has tough (high value) cases, and the cars that use those anti-theft devices? Well, they're probably a little more valuable than my 2003 Toyota Camry. (I can't give that thing away). We're probably comparing apples to oranges, and the failure to check (or even mention that as a possibility is definitely a flaw in the argument.

Furthermore, because I think it's worth noting for detail's sake, notice that a "theft" is not the same thing as "breaking into something." Now, we don't know, because the stimulus is officially like a fourth of the length of this explanation, but it could be the case that the whole point of the anti-theft is to lure thieves in, like a Venus fly trap, and then lock them inside while alerting the police. I will grant you that it does use the word "stolen," but that is immediately followed by an "or," so we don't really know what the statistic is actually capturing. And that really summarizes the flaw in the argument. It doesn't provide any evidence to truly support the weight of its rather bulky claim. As you'll learn in law school, this type of claim would have Summary Judgment Motion written all over it.

Let me know if you need further clarification. We didn't really talk about the answer choices, but hopefully why the argument is flawed (for the same reason the library to book purchase ratio claim in answer choice C is flawed) is clearer now.
User avatar
 catherineshi99
  • Posts: 9
  • Joined: May 23, 2021
|
#88919
That was an excellent explanation. Thank you!
User avatar
 annabelle.swift
  • Posts: 54
  • Joined: Sep 01, 2021
|
#92624
Hi, I got this question right, but I wanted to double-check my reasons for eliminating the other answer choices because they didn't quite match up with the administrator explanation:

A) Eliminated b/c its conclusion mentions a third idea (good schools) that was not present in its premise (which only mentions flourishing public libraries and better-educated citizens). The stimulus' conclusion does not bring in a third idea.
D) If this were to match the stimulus, its conclusion should have said something like "it follows that reading voraciously degrades one's vision."
E) Eliminated b/c the conclusion says that an entity "should" do a specific action, while the stimulus conclusion never recommends a specific action.
Is my reasoning solid? Thanks!
 Robert Carroll
PowerScore Staff
  • PowerScore Staff
  • Posts: 1787
  • Joined: Dec 06, 2013
|
#92634
annabelle.swift,

Your reasoning for answer choice (A) looks great.

Your reasoning for answer choice (D) is a bit off. The stimulus is concluding that there is NOT a causal relationship. So answer choice (D) would not have been closer to the stimulus if it had concluded that there was causation.

I think your reasoning for answer choice (E) is good - the normative language of this answer does not match the stimulus. You can supplement that with what the first post's explanation said and have even more reason for not picking this answer.

Robert Carroll

Get the most out of your LSAT Prep Plus subscription.

Analyze and track your performance with our Testing and Analytics Package.